Looking for infinite series resembling an exponential












3












$begingroup$


I'm looking for some $f(x)$ that has the following property:



$sum_{x=1}^infty f(kx) = r^k$



for some real $0 < r < 1$, and at least for strictly positive integer $k$.



Does such an $f(x)$ exist?



This could also be thought of in terms of some sequence of real numbers $f[n]$.



I posted this at MSE but got no answer, so thought I would try MO.










share|cite|improve this question









$endgroup$

















    3












    $begingroup$


    I'm looking for some $f(x)$ that has the following property:



    $sum_{x=1}^infty f(kx) = r^k$



    for some real $0 < r < 1$, and at least for strictly positive integer $k$.



    Does such an $f(x)$ exist?



    This could also be thought of in terms of some sequence of real numbers $f[n]$.



    I posted this at MSE but got no answer, so thought I would try MO.










    share|cite|improve this question









    $endgroup$















      3












      3








      3





      $begingroup$


      I'm looking for some $f(x)$ that has the following property:



      $sum_{x=1}^infty f(kx) = r^k$



      for some real $0 < r < 1$, and at least for strictly positive integer $k$.



      Does such an $f(x)$ exist?



      This could also be thought of in terms of some sequence of real numbers $f[n]$.



      I posted this at MSE but got no answer, so thought I would try MO.










      share|cite|improve this question









      $endgroup$




      I'm looking for some $f(x)$ that has the following property:



      $sum_{x=1}^infty f(kx) = r^k$



      for some real $0 < r < 1$, and at least for strictly positive integer $k$.



      Does such an $f(x)$ exist?



      This could also be thought of in terms of some sequence of real numbers $f[n]$.



      I posted this at MSE but got no answer, so thought I would try MO.







      real-analysis sequences-and-series power-series






      share|cite|improve this question













      share|cite|improve this question











      share|cite|improve this question




      share|cite|improve this question










      asked 3 hours ago









      Mike BattagliaMike Battaglia

      1,036523




      1,036523






















          1 Answer
          1






          active

          oldest

          votes


















          4












          $begingroup$

          You can solve this system of equations explicitly in terms of the function
          $$F(z)=sum_{m=1}^{infty} mu(m)z^m=z-z^2-z^3-z^5+z^6+cdots$$
          which can be shown to be transcendental (see the paper "Transcendence of Power Series for Some Number Theoretic Functions", by Coons and Borwein). With this notation your sequence can be succinctly expressed as
          $$f(n)=F(r^n).$$
          Indeed, one can check that
          $$sum_{n=1}^{infty} f(kn)=sum_{n=1}^{infty} F(r^{kn})=sum_{m,n=1}^{infty} mu(m)r^{nmk}=sum_{m=1}^{infty}sum_{d|m}mu(d)r^{mk}=r^k$$
          as desired.






          share|cite|improve this answer









          $endgroup$













            Your Answer





            StackExchange.ifUsing("editor", function () {
            return StackExchange.using("mathjaxEditing", function () {
            StackExchange.MarkdownEditor.creationCallbacks.add(function (editor, postfix) {
            StackExchange.mathjaxEditing.prepareWmdForMathJax(editor, postfix, [["$", "$"], ["\\(","\\)"]]);
            });
            });
            }, "mathjax-editing");

            StackExchange.ready(function() {
            var channelOptions = {
            tags: "".split(" "),
            id: "504"
            };
            initTagRenderer("".split(" "), "".split(" "), channelOptions);

            StackExchange.using("externalEditor", function() {
            // Have to fire editor after snippets, if snippets enabled
            if (StackExchange.settings.snippets.snippetsEnabled) {
            StackExchange.using("snippets", function() {
            createEditor();
            });
            }
            else {
            createEditor();
            }
            });

            function createEditor() {
            StackExchange.prepareEditor({
            heartbeatType: 'answer',
            autoActivateHeartbeat: false,
            convertImagesToLinks: true,
            noModals: true,
            showLowRepImageUploadWarning: true,
            reputationToPostImages: 10,
            bindNavPrevention: true,
            postfix: "",
            imageUploader: {
            brandingHtml: "Powered by u003ca class="icon-imgur-white" href="https://imgur.com/"u003eu003c/au003e",
            contentPolicyHtml: "User contributions licensed under u003ca href="https://creativecommons.org/licenses/by-sa/3.0/"u003ecc by-sa 3.0 with attribution requiredu003c/au003e u003ca href="https://stackoverflow.com/legal/content-policy"u003e(content policy)u003c/au003e",
            allowUrls: true
            },
            noCode: true, onDemand: true,
            discardSelector: ".discard-answer"
            ,immediatelyShowMarkdownHelp:true
            });


            }
            });














            draft saved

            draft discarded


















            StackExchange.ready(
            function () {
            StackExchange.openid.initPostLogin('.new-post-login', 'https%3a%2f%2fmathoverflow.net%2fquestions%2f321432%2flooking-for-infinite-series-resembling-an-exponential%23new-answer', 'question_page');
            }
            );

            Post as a guest















            Required, but never shown

























            1 Answer
            1






            active

            oldest

            votes








            1 Answer
            1






            active

            oldest

            votes









            active

            oldest

            votes






            active

            oldest

            votes









            4












            $begingroup$

            You can solve this system of equations explicitly in terms of the function
            $$F(z)=sum_{m=1}^{infty} mu(m)z^m=z-z^2-z^3-z^5+z^6+cdots$$
            which can be shown to be transcendental (see the paper "Transcendence of Power Series for Some Number Theoretic Functions", by Coons and Borwein). With this notation your sequence can be succinctly expressed as
            $$f(n)=F(r^n).$$
            Indeed, one can check that
            $$sum_{n=1}^{infty} f(kn)=sum_{n=1}^{infty} F(r^{kn})=sum_{m,n=1}^{infty} mu(m)r^{nmk}=sum_{m=1}^{infty}sum_{d|m}mu(d)r^{mk}=r^k$$
            as desired.






            share|cite|improve this answer









            $endgroup$


















              4












              $begingroup$

              You can solve this system of equations explicitly in terms of the function
              $$F(z)=sum_{m=1}^{infty} mu(m)z^m=z-z^2-z^3-z^5+z^6+cdots$$
              which can be shown to be transcendental (see the paper "Transcendence of Power Series for Some Number Theoretic Functions", by Coons and Borwein). With this notation your sequence can be succinctly expressed as
              $$f(n)=F(r^n).$$
              Indeed, one can check that
              $$sum_{n=1}^{infty} f(kn)=sum_{n=1}^{infty} F(r^{kn})=sum_{m,n=1}^{infty} mu(m)r^{nmk}=sum_{m=1}^{infty}sum_{d|m}mu(d)r^{mk}=r^k$$
              as desired.






              share|cite|improve this answer









              $endgroup$
















                4












                4








                4





                $begingroup$

                You can solve this system of equations explicitly in terms of the function
                $$F(z)=sum_{m=1}^{infty} mu(m)z^m=z-z^2-z^3-z^5+z^6+cdots$$
                which can be shown to be transcendental (see the paper "Transcendence of Power Series for Some Number Theoretic Functions", by Coons and Borwein). With this notation your sequence can be succinctly expressed as
                $$f(n)=F(r^n).$$
                Indeed, one can check that
                $$sum_{n=1}^{infty} f(kn)=sum_{n=1}^{infty} F(r^{kn})=sum_{m,n=1}^{infty} mu(m)r^{nmk}=sum_{m=1}^{infty}sum_{d|m}mu(d)r^{mk}=r^k$$
                as desired.






                share|cite|improve this answer









                $endgroup$



                You can solve this system of equations explicitly in terms of the function
                $$F(z)=sum_{m=1}^{infty} mu(m)z^m=z-z^2-z^3-z^5+z^6+cdots$$
                which can be shown to be transcendental (see the paper "Transcendence of Power Series for Some Number Theoretic Functions", by Coons and Borwein). With this notation your sequence can be succinctly expressed as
                $$f(n)=F(r^n).$$
                Indeed, one can check that
                $$sum_{n=1}^{infty} f(kn)=sum_{n=1}^{infty} F(r^{kn})=sum_{m,n=1}^{infty} mu(m)r^{nmk}=sum_{m=1}^{infty}sum_{d|m}mu(d)r^{mk}=r^k$$
                as desired.







                share|cite|improve this answer












                share|cite|improve this answer



                share|cite|improve this answer










                answered 3 hours ago









                Gjergji ZaimiGjergji Zaimi

                62.8k4163310




                62.8k4163310






























                    draft saved

                    draft discarded




















































                    Thanks for contributing an answer to MathOverflow!


                    • Please be sure to answer the question. Provide details and share your research!

                    But avoid



                    • Asking for help, clarification, or responding to other answers.

                    • Making statements based on opinion; back them up with references or personal experience.


                    Use MathJax to format equations. MathJax reference.


                    To learn more, see our tips on writing great answers.




                    draft saved


                    draft discarded














                    StackExchange.ready(
                    function () {
                    StackExchange.openid.initPostLogin('.new-post-login', 'https%3a%2f%2fmathoverflow.net%2fquestions%2f321432%2flooking-for-infinite-series-resembling-an-exponential%23new-answer', 'question_page');
                    }
                    );

                    Post as a guest















                    Required, but never shown





















































                    Required, but never shown














                    Required, but never shown












                    Required, but never shown







                    Required, but never shown

































                    Required, but never shown














                    Required, but never shown












                    Required, but never shown







                    Required, but never shown







                    Popular posts from this blog

                    SQL Server 17 - Attemping to backup to remote NAS but Access is denied

                    Always On Availability groups resolving state after failover - Remote harden of transaction...

                    Restoring from pg_dump with foreign key constraints